www.vorhilfe.de
- Förderverein -
Der Förderverein.

Gemeinnütziger Verein zur Finanzierung des Projekts Vorhilfe.de.
Hallo Gast!einloggen | registrieren ]
Startseite · Mitglieder · Impressum
Forenbaum
^ Forenbaum
Status VH e.V.
  Status Vereinsforum

Gezeigt werden alle Foren bis zur Tiefe 2

Navigation
 Startseite...
 Suchen
 Impressum
Das Projekt
Server und Internetanbindung werden durch Spenden finanziert.
Organisiert wird das Projekt von unserem Koordinatorenteam.
Hunderte Mitglieder helfen ehrenamtlich in unseren moderierten Foren.
Anbieter der Seite ist der gemeinnützige Verein "Vorhilfe.de e.V.".
Partnerseiten
Weitere Fächer:

Open Source FunktionenplotterFunkyPlot: Kostenloser und quelloffener Funktionenplotter für Linux und andere Betriebssysteme
Forum "Integration" - Integralrekursionsgleichung
Integralrekursionsgleichung < Integration < Funktionen < eindimensional < reell < Analysis < Hochschule < Mathe < Vorhilfe
Ansicht: [ geschachtelt ] | ^ Forum "Integration"  | ^^ Alle Foren  | ^ Forenbaum  | Materialien

Integralrekursionsgleichung: Frage (beantwortet)
Status: (Frage) beantwortet Status 
Datum: 02:29 So 29.11.2009
Autor: T_sleeper

Aufgabe
Bestimmen Sie [mm] I_{n}=\int_{0}^{\infty}x^{2n+1}e^{-ax^{2}},\, a>0,\, n\in\mathbb{N}_{0}, [/mm] indem Sie eine Rekursionsgleichung herleiten und [mm] I_0 [/mm] explizit bestimmen.

Hallo,

ich weiß hier nicht wie man diese Rekursionsgleichung herleitet. Das muss wohl etwas mit partieller Integration zu tun haben.
Muss ich dann am Ende eine Formel für [mm] I_{n-1} [/mm] oder eher [mm] I_{k+1}, k\in{0,...,n-1} [/mm] herausbekommen?
Und das wichtigste: Wie setze ich an bzw. gehe ich vor, um das herzuleiten?

Gruß Sleeper

        
Bezug
Integralrekursionsgleichung: Antwort
Status: (Antwort) fertig Status 
Datum: 02:38 So 29.11.2009
Autor: felixf

Hallo Sleeper!

> Bestimmen Sie [mm]I_{n}=\int_{0}^{\infty}x^{2n+1}e^{-ax^{2}},\, a>0,\, n\in\mathbb{N}_{0},[/mm]
> indem Sie eine Rekursionsgleichung herleiten und [mm]I_0[/mm]
> explizit bestimmen.
>  
> ich weiß hier nicht wie man diese Rekursionsgleichung
> herleitet. Das muss wohl etwas mit partieller Integration
> zu tun haben.

Ja, hat es. Schreibe $u' = [mm] x^{2 n + 1}$ [/mm] und $v = [mm] e^{-a x^2}$ [/mm] und leg los.

>  Muss ich dann am Ende eine Formel für [mm]I_{n-1}[/mm] oder eher
> [mm]I_{k+1}, k\in{0,...,n-1}[/mm] herausbekommen?

Du musst einen Ausdruck mit [mm] $I_{n + 1}$ [/mm] herausbekommen, wenn du einmal partiell integrierst. Dann loest du dies nach [mm] $I_{n+1}$ [/mm] auf.

>  Und das wichtigste: Wie setze ich an bzw. gehe ich vor, um
> das herzuleiten?

Siehe oben: du machst partielle Integration.

LG Felix


Bezug
                
Bezug
Integralrekursionsgleichung: Vernetzung ;-)
Status: (Mitteilung) Reaktion unnötig Status 
Datum: 02:42 So 29.11.2009
Autor: reverend

Die Aufgabe kam mir so bekannt vor...
guckstu hier.

Bezug
                
Bezug
Integralrekursionsgleichung: Frage (beantwortet)
Status: (Frage) beantwortet Status 
Datum: 03:12 So 29.11.2009
Autor: T_sleeper

Gut, ich bin nun soweit:
[mm] I_{n}=\int_{0}^{\infty}x^{2n+1}e^{-ax^{2}}dx=\left[\frac{x^{2n+2}}{2(n+1)}\cdot e^{-ax^{2}}\right]_{0}^{\infty}-\int_{0}^{\infty}\frac{x^{2n+2}}{2(n+1)}\cdot\left(-2axe^{-ax^{2}}\right)dx [/mm]
[mm] =\left[\frac{x^{2n+2}}{2(n+1)}\cdot e^{-ax^{2}}\right]_{0}^{\infty}+\frac{a}{n+1}\int_{0}^{\infty}x^{2n+3}e^{-ax^{2}}\, [/mm] dx

was stimmen müsste, wenn ich mich nicht verrechnet habe.
Das letzte Integral sieht nun schon ziemlich stark nach [mm] I_{n+1} [/mm] aus, aber so ganz passt es noch nicht.
Aber selbst wenn es das ist, habe ich in der Rekursionsformel doch dann auch wieder ein Integral drin oder nicht? Dann bringt mir das doch eigtl nicht wirklich etwas.


Bezug
                        
Bezug
Integralrekursionsgleichung: Antwort
Status: (Antwort) fertig Status 
Datum: 04:07 So 29.11.2009
Autor: felixf

Hallo!

> Gut, ich bin nun soweit:
>  
> [mm]I_{n}=\int_{0}^{\infty}x^{2n+1}e^{-ax^{2}}dx=\left[\frac{x^{2n+2}}{2(n+1)}\cdot e^{-ax^{2}}\right]_{0}^{\infty}-\int_{0}^{\infty}\frac{x^{2n+2}}{2(n+1)}\cdot\left(-2axe^{-ax^{2}}\right)dx[/mm]
>  
> [mm]=\left[\frac{x^{2n+2}}{2(n+1)}\cdot e^{-ax^{2}}\right]_{0}^{\infty}+\frac{a}{n+1}\int_{0}^{\infty}x^{2n+3}e^{-ax^{2}}\,[/mm]
> dx

Ja, das sieht doch gut aus. Warum rechnest du den Teil in den eckigen Klammern nicht mal aus?

> was stimmen müsste, wenn ich mich nicht verrechnet habe.
>  Das letzte Integral sieht nun schon ziemlich stark nach
> [mm]I_{n+1}[/mm] aus, aber so ganz passt es noch nicht.

Inwiefern passt es denn nicht?

>  Aber selbst wenn es das ist, habe ich in der
> Rekursionsformel doch dann auch wieder ein Integral drin
> oder nicht?

Noe, wo sollte denn ein Integral herkommen?

Rechne doch erstmal weiter, also mach den Klammerausdruck weg, setz [mm] $I_{n+1}$ [/mm] fuer das (passende) Integral ein, und loes das ganze nach [mm] $I_{n+1}$ [/mm] auf.

LG Felix


Bezug
                                
Bezug
Integralrekursionsgleichung: Frage (beantwortet)
Status: (Frage) beantwortet Status 
Datum: 15:53 So 29.11.2009
Autor: T_sleeper

Ich glaub ich habs nun.
Weitergerechnet siehts so aus:
[mm] =&\left[\frac{x^{2n+2}}{2(n+1)}\cdot e^{-ax^{2}}\right]_{0}^{\infty}+\frac{a}{n+1}\int_{0}^{\infty}x^{2n+3}e^{-ax^{2}}\, dx\\=&\underset{x\rightarrow\infty}{\mbox{lim}}\frac{x^{2n+2}}{2(n+1)}\cdot e^{-ax^{2}}+\frac{a}{n+1}\int_{0}^{\infty}x^{2n+3}e^{-ax^{2}}\, dx\\=&\frac{a}{n+1}\cdot I_{n+1} [/mm]
[mm] \Rightarrow I_{n+1}&=&I_{n}\cdot\frac{n+1}{a}. [/mm]

Passt es?
Naja um [mm] I_0 [/mm] ausrechnen zu können muss ich ja nur in meine Ausgangsformel n=0 setzen und integrieren, oder?

Die Aufgabe war ja eigentlich [mm] I_n [/mm] auszurechnen. Wenn ich nun also die Rekursionsformel hergeleitet hab und [mm] I_0 [/mm] berechnet hab, wie kann ich dann wirklich [mm] I_n [/mm] ausrechnen?

Bezug
                                        
Bezug
Integralrekursionsgleichung: Antwort
Status: (Antwort) fertig Status 
Datum: 17:40 So 29.11.2009
Autor: valoo

Ja, das passt so. Und [mm] I_{n} [/mm] kann man nur rekursiv berechnen. Was willst du da anders machen? Für [mm] I_{0} [/mm] integrierst du das einfach und da sollte [mm] \bruch{1}{2*a} [/mm] rauskommen.

Bezug
                                        
Bezug
Integralrekursionsgleichung: Antwort
Status: (Antwort) fertig Status 
Datum: 22:58 So 29.11.2009
Autor: felixf

Hallo!

> Ich glaub ich habs nun.
>  Weitergerechnet siehts so aus:
>  [mm]=&\left[\frac{x^{2n+2}}{2(n+1)}\cdot e^{-ax^{2}}\right]_{0}^{\infty}+\frac{a}{n+1}\int_{0}^{\infty}x^{2n+3}e^{-ax^{2}}\, dx\\=&\underset{x\rightarrow\infty}{\mbox{lim}}\frac{x^{2n+2}}{2(n+1)}\cdot e^{-ax^{2}}+\frac{a}{n+1}\int_{0}^{\infty}x^{2n+3}e^{-ax^{2}}\, dx\\=&\frac{a}{n+1}\cdot I_{n+1}[/mm]
>  
> [mm]\Rightarrow I_{n+1}&=&I_{n}\cdot\frac{n+1}{a}.[/mm]
>  
> Passt es?

Sieht gut aus.

>  Naja um [mm]I_0[/mm] ausrechnen zu können muss ich ja nur in meine
> Ausgangsformel n=0 setzen und integrieren, oder?

Ja.

> Die Aufgabe war ja eigentlich [mm]I_n[/mm] auszurechnen. Wenn ich
> nun also die Rekursionsformel hergeleitet hab und [mm]I_0[/mm]
> berechnet hab, wie kann ich dann wirklich [mm]I_n[/mm] ausrechnen?

Nun, du kannst die Rekursionsgleichung versuchen aufzuloesen.

Es ist ja [mm] $I_n [/mm] = [mm] \frac{n}{a} I_{n-1} [/mm] = [mm] \frac{n (n - 1)}{a^2} I_{n-2} [/mm] = [mm] \frac{n (n - 1) (n - 2)}{a^3} I_{n-3}$. [/mm] Man kann also vermuten, dass [mm] $I_n [/mm] = [mm] \frac{n!}{a^n} I_0$ [/mm] ist. Das kannst du jetzt mal versuchen per Induktion nachzuweisen.

LG Felix


Bezug
Ansicht: [ geschachtelt ] | ^ Forum "Integration"  | ^^ Alle Foren  | ^ Forenbaum  | Materialien


^ Seitenanfang ^
ev.vorhilfe.de
[ Startseite | Mitglieder | Impressum ]